Vous êtes sur la page 1sur 60

************* DATA SUFFICIENCY *********

DIRECTIONS
Each of the problems below consists of a question and two statements, labelled
(1) and (2), in which certain data are given.
You must decide whether the data given in the statements are sufficient to answer
the question.
Using the data given in the statements plus your knowledge of mathematics and
everyday facts (such as the number of days in July or the meaning of counter
clockwise), you are to answer.
^
(A) if statement (1) ALONE is sufficient, but statement (2) alone is not sufficient
to answer the question asked;
(B) if statement (2) ALONE is sufficient, but statement (1) alone is not sufficient
to answer the question asked;
(C) if BOTH statements (1) and (2) TOGETHER are sufficient to answer the question
asked, but NEITHER statement ALONE is sufficient;
(D) if EACH statement ALONE is sufficient to answer the question asked;
(E) if statements (1) and (2) TOGETHER are NOT sufficient to answer the question
asked, and additional data specific to the problem are needed.
^
Example
1. How many rectangular tiles are required for the kitchen floor?
(1) Each tile is 48 square inches.
(2) The kitchen measures 10 feet by 7 feet.
^

#1
@gmat1.dll 101 100 150
4
~
What is the perimeter of triangle MNO?
~
(1) The area of triangle MNO is 24.
(2) Angle MON equals 90 degree.
%
(1) tells us the area of triangle MNO is 24, its height must be 6. Note the side MO
will be the height because 6-8-10 is a Pythagorean triple. Thus we can find the
perimeter. Using statement (2) we know triangle MON is a right triangle and can use
the Pythagorean theorem to find the third side.
%
#2
@gmat1.dll 102 100 100
5
~
What is the length of side c?
~
(1) a = 6
(2) b = 7
%
The Pythagorean theorem a x a + b x b = c x c, can be employed only for a triangle
known to have a right angle.
%
#3
@gmat1.dll 103 100 100
2
~
Within the circle with centre C, how many degrees is angle ABC?
~
(1) Angle CBD = 31 degree
(2) Angle BDA = 48 degree
%
The central angle ACB must be 96 degree, twice the inscribed angle BDA. Further ,
the triangle ABC has two radii as sides, so it is isosceles, and angle CAB = ABC.
Therefore 2(ABC)+96 degree = 180 degree, and one may solve for ABC = 42 degree.
%
#4
@gmat1.dll 104 100 100
1
~
How many degrees in angle q?
~
(1) m + p = 82 degrees
(2) n + p = 111 degrees
%
Because vertical angles are equal, p = t. From statement (1) we can find that
angle p = 41 degrees. In the figure, angles p and t are supplementary, so angle t
= 180 - 41 = 139 degrees.
%
#5
@gmat1.dll 105 100 100
4
~
How long is KL?
~
(1) ML = 6+
(2) KN = 6
%
Either statement suffices to determine KL to be 7+. Because of similar triangles,
KL:MO = KN:NO = LN:MN. From the Pythagorus theorem, MN = 13. By addition LN =
LM+MN = 6+ + 13 = 19+, also KN = 18. Therefore KL:5 = 18:12 = 19+:13 and we can
solve for KL.
%
#6
@gmat1.dll 106 100 100
4
~
What is the width AB of rectangle ABCD?
~
(1) CD = x + 4
(2) The area is 48
%
From the first statement, x+4 = 4x -2, because opposite sides of a rectangle are
equal. So x = 2 and the width x +4 = 6. Or we can use the area of the second
statement, as width =area / length = 48 / 8 = 6.
%
#7
@gmat1.dll 107 100 100
5
~
What is the diameter of the wheel above?
~
(1) The wheel has rolled 36 feet.
(2) Point P has touched the ground 5 times.
%
Since P is not now touching the ground, all we know is that the wheel has travelled
more than 5 circumferences but less than 5+. So, 5PId < 36 <5+PId. Hence the
diameter is uncertain, somewhere in the range 2.18 to 2.29 feet.
%
#8
@gmat1.dll 108 100 100
2
~
Given QS perpendicular to PR, what is the area of triangle QRS?
~
(1) Angle QPS = 69 degree
(2) QS = RS
%
From the Pythagorean theorem we know that PSxPS + QSxQS = PQxPQ, so QSxQS = 16
+121, and QS = +137. The second statement tells us that the base and height of
the triangle are equal.
%
#9
@gmat1.dll 109 100 100
1
~
What is the area of the shaded portion of the half-circle?
~
(1) Q is the midpoint of PQ.
(2) Angle PQR is a right angle.
%
Angle PQR, inscribed on a diameter, is necessarily 90 degree, so the second
statement adds nothing new. But the first statement implies the triangle has an
altitude equal to the circle's radius 6. So the shaded area equals the area of
the semicircle minus the area of the triangle.
%
#10
@gmat1.dll 110 100 100
5
~
In the circle with centre R above, what is the area of right triangle PRT?
~
(1) PT equals 16 inches.
(2) QS equals 12 inches.
%
The area of the right triangle equals half the product of the two legs, PT and RT.
The length of PT is given(1). But point T is not on the circle, so RT is less than
(QS), the diameter (2). Lacking the length of RT, one cannot calculate the area.
%
#11
@gmat1.dll 111 100 100
4
~
How many degrees in angle a?
~
(1) b = 47 degrees
(2) c = 110 degrees
%
From the first statement, a = b = 47 degree, for vertical angles are equal. From
the second statement, the interior angle adjacent to c is 70 degree, because that
angle and c sum to a straight angle. Since the interior angles of a triangle add
to 180 degree, you can write the equation a + 63 degree + 70 degree = 180 degree,
permitting the determination of a.
%
#12
@gmat1.dll 112 100 100
3
~
In the figure above, PQRS is a rectangle and T is the midpoint of one side. What
is the area of triangle QRS?
~
(1) QT = 5 inches
(2) RS = 3 inches
%
In the rectangle QP = SR = 3 inches. So two sides of right triangle QPT are known
and the length of side PT, can be found with the Pythagorean theorem to be 4
inches. As point T is the midpoint of PS, the length of the rectangle is twice PT,
or 8 inches. The area of the right triangle QRS is half the product of the sides
adjacent to the right angle :
P = (1/2)bh = (1/2)(8)(3) = 12 sq. in.
%
#13
@gmat1.dll 113 100 100
1
~
What is the ratio of the area of quadrilateral NOPQ to the area of triangle MNQ?
~
(1) NO = (1/2)MO
(2) Angle ONQ = 135 degree
%
Area NOPQ equals triangle MOP minus triangle MNQ. From the sketch it is clear that
the two triangles are similar, as their respective sides are parallel. From (1) we
see that the base and height of MNQ are half those of MOP, hence the area of MNQ is
one quarter that of MOP. Consequently, the area NOPQ is three times the area of
MNQ.
%
#14
@gmat1.dll 114 100 100
4
~
In rhombus PQRS, what is angle QRS?
~
(1) QR = QS
(2) Angle QSP = 60 degree
%
The definition of a rhombus stipulates the equality of its four sides. The first
statement reveals that the two triangles are equilateral, so all the angles must be
60 deg. From the second statement, RQS = 60 deg (by alternate interior angles),
triangle QRS is isosceles with QR = RS (from the definition of a rhombus), so angle
QSR = RQS = 60 deg, subtracting those angles from 180 deg leaves 60 deg for angle
QRS.
%
#15
@gmat1.dll 115 100 100
2
~
How many degrees in angle r?
~
(1) q is a right angle.
(2) p + q = 131 deg
%
Since the three angles sum to a straight angle, r = 180 - 131 = 49 deg.
%
#16
@gmat1.dll 116 100 100
4
~
Given the AB is tangent to the circle with centre C, what is the length of the
circle's radius?
~
(1) AD = 4
(2) AC = 10
%
The radius BC must be perpendicular to the tangent AB so ABC is a right triangle.
The statement implies it is a 6-8-10 right triangle, so the radius is 6. We can
solve from the first statement with the Pythagorean theorem.
#17
@gmat1.dll 117 100 100
2
~
How many degrees is angle MON?
~
(1) Angle NOP = 35.
(2) MO is perpendicular to ON.
%
Since MO is perpendicular to ON, MON is a right angle.
%
#18
@gmat1.dll 118 100 100
1
~
The centres of the 3 circles lie on one line (the circles intersect as shown).
What is the area of the shaded part of the figure?
~
(1) Each of the small circles has a diameter of 5.
(2) The large circle has a radius of 5.
%
The shaded part equals to the area of the large circle minus the areas of the two
small circles. For any circle, A = pi x r x r and we can ascertain the radii of
the circles from statement (1) alone. The small circles are of equal size, with
radii of 2.5. Because the small circles are equal, their diameter is the radius of
the large circle.
%
#19
@gmat1.dll 119 100 100
3
~
In the trapezoid MNOP, what is the area of triangle MNO?
~
(1) MO = 10
(2) NO = 7
%
In the trapezoid, NO is evidently parallel to MP. The first statement reveals that
triangle MOP is a 6-8-10 right triangle, with angle MPO = 90 deg . Because NO is
parallel to MP, angle NOP must also equal 90 deg . Consider triangle MNO with base
NO. The altitude to vertex M equals OP, because OP is perpendicular to NO. Then
the area = (1/2)bh = (1/2)(NO)(OP) = (1/2) x 7 x 6 = 21
%
#20
@gmat1.dll 120 100 100
1
~
Is the line AB parallel to the line DC?
~
(1) x = y
(2) q = r
%
Since x,y are alternate interior angles, if STATEMENT(1) is true AB is parallel to
CD.
So (1) alone is sufficient. Thus, the answer to question I is YES and the only
possible choices are A and D. Statement(2) alone is not sufficient since the line
CD can be moved so that q is still equal to r but AB and CD are not parallel.
Therefore, the answer to question II is NO, and the correct choice is A.
%
#21
@gmat1.dll 121 100 100
1
~
Which triangle, PST or PTR, has the larger area? PQRS is a rectangle.
~
(1) ST is longer than TR.
(2) PR is longer than PT.
%
Since the area of a triangle is 1/2(altitude)(base) and since both triangles have
SP as an altitude, if the base (ST) of triangle PST is larger than the base (TR) of
triangle PTR, then the area of PST is larger than the area of PTR. So (1) alone is
sufficient, and the answer to question I is YES.
(2) alone is not sufficient since for any point T between S and R (2) will be true,
but, depending on whether T is closer to S or R, a different triangle will have
the larger area. So the answer to question II is NO, and the correct choice is A.
%
#22
@gmat1.dll 122 100 100
4
~
ABCDEFGH is a cube. What is the length of the line segment AG?
~
(1) the length of the line segment AB is 4 inches.
(2) the area of the square BCGH is 16 square inches.
%
By using the distance formula (Pythagorean theorem) you could find the length of AG
if you knew the lengths of AH and GH (or if you knew the lengths of AC and CG or
many other combinations). If you knew the lengths of AB and BH, then you could
find the length of AH. Thus, it is sufficient to know the lengths of AB, BH, and
GH all have same length since they are all edges of the cube. So it is sufficient
to know the length of an edge of the cube. Now statements (1) and (2) alone are
sufficient, and the correct choice is D. Notice that, if you knew that (1) and (2)
are equivalent, then the only possible choices are D or E, so you can make an
intelligent guess.
%
#24
@gmat1.dll 124 100 100
4
~
In triangle PQR, find c if PQ = 5 and b = 40.
~
(1) QR = 5
(2) The bisector of angle Q is perpendicular to PR.
%
(1) alone is sufficient since QR = PQ implies a-b = 40. Since the sum of the
angles in a triangle is 180 degree, c must equal to 100 degree. (2) alone is
sufficient. Let T be the point where the bisector of angle Q meets PR. Then
according to (2), triangle QTR is a right angle. Since angle b is 40 degree, the
remaining angle in triangle QTR is 50 degree and equals (1/2)c, so c = 100 degree.
%
#25
@gmat1.dll 125 100 100
5
~
Is the figure PQRS a rectangle?
~
(1) a = 90
(2) PQ = RS.
%
If PQRS has the pairs of opposite sides equal and each angle is 90 degree, then it
is a rectangle. But there are many quadrilaterals that have two opposite sides
equal with one angle a right angle. Therefore (1) and (2) together are
insufficient
%
#26
@gmat1.dll 126 100 100
3
~
Is angle QPR a right angle ?
~
(1) a = 2b.
(2) b = 1.5c.
%
Since the sum of the angles in a triangle is 180 deg, a+b+c = 180. Using (1) alone
we have 2b +b+ c = 3b +c = 180, which is insufficient to determine b or c. Using
(2) alone we have a+1.5c+c = a+2.5c = 180, which is not sufficient to determine a
or c. However, if we use both (1) and (2) we obtain 3b + c = 4.5c + c = 5.5c =
180, so c = 2 /11 of 180. Now b = 3/2 of c, so b = 3/11 of 180, and a = 6/11 of
180. Therefore, angle QPR is not a right angle and (1) and (2) are sufficient.
%
#27
@gmat1.dll 127 100 100
1
~
The pentagon PQRST is inscribed in the circle with centre O. How many degrees is
angle PQR?
~
(1) The pentagon PQRST is a regular pentagon.
(2) The radius of the circle is 5 inches.
%
The sum of the angles of the pentagon is 540 deg. ( The sum of the angles of a
polygon with n sides which is inscribed in a circle is (n-2)180 deg.) Statement
(1) alone is sufficient. If the polygon is regular, all angles are equal and so
angle PQR is 1/5 of 540 deg or 108 deg. Statement (2) alone is insufficient
because the radius of the circle does not give any information about the angles of
the pentagon
%
#28
@gmat1.dll 128 100 100
2
~
What is the area of the circle with centre O? (PQ and XY are straight lines)
~
(1) XY = 5 inches
(2) PQ = 7 inches
%
The area of a circle is Pixrxr, where r is the radius of the circle. Since O is a
point on the line PQ, PQ is a diameter of the circle. Therefore, since a radius is
one half of a diameter, the radius of the circle is 3.5 inches. Thus (2) alone is
sufficient. (1) alone is insufficient since there is no relation between XY and the
radius.
%
#29
@gmat1.dll 129 100 100
2
~
Is PQ is perpendicular to RS?
~
(1) PR = QS
(2) a = b
%
Statement (1) alone is not sufficient. If the segment PR is moved further away
from the segment QT, then the angles a and b will change. So statement (1) does
not ensure that RS and PQ are perpendicular. Statement (2) alone is sufficient.
Since PQ is a straight line, a+b equals 180. Thus, if a = b, a and b both equal 90
and PQ is perpendicular to RS. So the correct answer is B.
%
#30
@gmat1.dll 130 100 100
3
~
The hexagon PQRSTU is inscribed in the circle with centre O. What is the length of
PQ?
~
(1) The radius of the circle is 4 inches.
(2) The hexagon is a regular hexagon
%
Draw the radii from O to each of the vertices. These lines divide the hexagon into
six triangles. Statement (2) says that all the triangles are congruent since each
of their pairs of corresponding sides is equal. Since each of their pairs of
corresponding sides is equal. Since there are 360 degree in a circle, the central
angle of each triangle is 60 degree. And since all radii are equal, each angle of
the triangle equals 60 degree. Therefore, the triangles are equilateral, and PQ is
equal to the radius of the circle. Thus, if we assume Statement (1), we know the
length of PQ. Without statement (1) we can't find the length PQ. Also statement
(1) alone is not sufficient, since PQ need not equal the radius unless the hexagon
is regular.
%
#31
@gmat1.dll 131 100 100
1
~
Is PQR a right triangle ? PQ = 5 , PR = 4
~
(1) QR = 3
(2) PR = RS
%
Statement (1) alone is sufficient. Since 3x3 + 4x4 = 5x5, PQR is a right triangle
by the Pythagorean theorem. Statement (2) alone is not sufficient since you can
choose a point S so that PR = RS for any triangle PQR.
%
#32
@gmat1.dll 132 100 100
1
~
Is PQ greater than PR?
~
(1) a > b
(2) PR > PS
%
Statement (1) alone is sufficient. If b > a then the side opposite angle PQR is
larger than the side opposite angle PRQ. (2) alone is insufficient, since S can be
anywhere between Q and R, so you can't decide whether PS is larger or smaller than
PQ.
%
#33
@gmat1.dll 133 100 100
5
~
What is the area of the circular section XOY? X and Y are points on the circle
which has O as its centre.
~
(1) Angle XOY = 36 deg
(2) OY = OX
%
Since the area of a circle is pixrxr, the area of the circular section XOY is the
fraction x / 360 times pixrxr, where angle XOY = x degree. (There are 360 deg in
the entire circle) Using statement (1), we know x = 36 so (x/360)pixrxr =
(1/10)pixrxr. However, statement (1) gives no information, about the value of r,
so statement (1) alone is insufficient. Statement (2) gives no information about
the value of r . So statements (1) and (2) together are insufficient.
%
#34
@gmat1.dll 134 100 100
3
~
What is the area of the shaded part of the circle? O is the centre of the circle.
~
(1) The radius of the circle is 4.
(2) a is 60.
%
Statement (1) tells us the area of the circle is pix4x4 = 16pi. Since there are
360 deg in the whole circle, (2) tells us that the shaded area is 60 / 360 or 1/ 6
of the area of the circle. Thus using both (1) and (2) of the area of the circle.
Thus, using both (1) and (2), we can answer the question, but since we need both
the radius of the circle and the value of a, neither of them alone is sufficient .
Therefore answer is (C).
%
#35
@gmat1.dll 135 100 100
4
~
If a and a' are straight lines, find n.
~
(1) m= 100
(2) o = 80
%
Since a straight line forms an angle 180 deg and a' is a straight line, we know m
+ n = 180. If we use statement (1) we get n = 80, so (1) alone is sufficient.
When two straight lines intersect, the vertical angles are equal. So n = o, thus
if we use (2) we get n = 80. Therefore, (2) alone is sufficient. thus, each
statement alone is sufficient.
%
#36
@gmat1.dll 136 100 100
3
~
Fifty students have signed up for at least one of the courses German I and English
I. How many of the 50 students are taking German I but not English I.
~
(1) 16 students are taking German I and English I.
(2) No of students taking English I but not German I is the same as the no taking
German I but not English I.
%
In the figure, a denotes the number taking German I but not English I, and b the
number taking English I but not German I. From (1) we know that a + 16 + b = 50,
from (2) a = b. Neither statement alone can be solved for a, but both together are
sufficient (and yield a = 17).
%
#37
@gmat1.dll 137 100 100
5
~
Is PQRS a square?
~
(1) PS = PQ
(2) a = 90
%
Statement (1) alone is not sufficient because it only says two sides are equal, in
a square all four sides are equal. Even if we use (2) we don't know if PQRS is a
square since all angles have to be right angles in a square. Therefore, both
statements together are insufficient. Another way to solve this problem would be
to visualise what happens to figure PQRS if R is moved while P, Q, and S are kept
the same. (1) and (2) would still hold even when R is moved, but the figure PQRS
could change from a square to a non-square. Thus (1) and (2) together are not
sufficient.
%
#38
@gmat1.dll 138 100 100
3
~
a, a', and b are straight lines. Are a and a' parallel?
~
(1) x = y
(2) y = z
%
Statement (1) alone is insufficient. If x and y were right angles, (1) would imply
that a and a' are parallel but if x and y are not right angles (1) would imply that
a and a' are not parallel. (2) alone is not sufficient since it gives information
only about a' and says nothing about the relation of a and a' . (1) and (2)
together give x = z which means that a and a' are parallel. Therefore, (1) and (2)
together are sufficient but neither alone is sufficient.
%
#39
@gmat1.dll 139 100 100
5
~
Which of the two figures, PQRS or LMNO, has the largest area?
~
(1) The perimeter of PQRS is longer than the perimeter of LMNO.
(2) PR is longer than LN.
%
Statement (1) alone is not sufficient. A four sided figure can have both large
perimeter and smaller area than another four sided figure or it could have larger
perimeter and larger area. (2) alone is also insufficient since the length of one
diagonal does not determine the area of a four-sided figure. (1) and (2) together
are also insufficient, so the answer is (E).
%
#40
@gmat1.dll 140 100 100
1
~
PQRS is a rectangle. Which region, PQTU or RSUT, has a larger area?
~
(1) QT is longer than SU
(2) QT is longer than RS
%
Both regions PQTU and RSUT are trapezoids, so their area is given by the formula
a(0.5(b1+b2 )) where a is an altitude and b1 and b2 are the sides perpendicular to
the altitude. Because PQRS is a rectangle, PQ = RS, which means the altitudes are
the same length for each region. So it is sufficient to know whether QT+ PU is
larger than TR + US. Statement (1) alone is sufficient, since if QT is larger than
US, then QR -QT, which is TR must be smaller than PS - US = PU.(PS = QR since
PQRS is a rectangle) So QT +PU is larger than TR+US . Statement (2) alone is not
sufficient, since either region could be larger if QT is larger than RS.
#41
@gmat1.dll 141 100 100
4
~
PQRS is a square. QRO is a semicircle. What is the area of PQORS?
~
(1) The length of PR is 4 x sq rt 2
(2) The radius of the semicircle QOR is 2.
%
The area of the region is the area of the square plus the area of the semicircle.
So you must be able to determine the length of a side of the square and the length
of the radius of the semicircle. Since the radius is 1/2 of QR, it is sufficient
to determine either the radius or the length of a side of the square. Statement(1)
alone is sufficient, since the diagonal of a square is sq rt 2 times the length of
a side. Statement (2) alone is sufficient, since the length of a side of the
square is twice the radius.
#42
@gmat1.dll 142 100 100
3
~
Is the above figure a cube?
~
(1) The lengths of all edges are equal.
(2) The angle between any two edges that meet is a right angle.
%
A cube is a solid with 6 faces, all of which are congruent squares and any two
faces that intersect are perpendicular. Statement (1) is not sufficient since a
solid with 2 of the faces as diamonds is not a cube but does satisfy (1).
Statement (1) is not sufficient since a solid with 2 or 4 of the faces as congruent
rectangles is not a cube. But (1) and (2) together mean that each face is a
congruent square.
#43
@gmat1.dll 143 100 100
4
~
Find the length of PR if PQ has length 3 and a is 45.
~
(1) c = 45
(2) b = 90
%
Since we know that the sum of the angles in a triangle is 180 deg and that a = 45,
statement (1) implies statement (2) and (2) implies (1). Either one is
sufficient, since if c = 45, then a = c and the sides opposite the equal angles are
equal. Hence PR = PQ = 3.
#44
@gmat1.dll 144 100 100
3
~
Is PQRS a rectangle ?
~
(1) PR and QS bisect each other at O
(2) Angle PSR is 90 deg
%
Statement (1) is not sufficient since the diagonals of any parallelogram bisect
each other. Statement (2) is not sufficient since the other angles of the figure
do not have to be right angles. However, (1) and (2) together are sufficient.
Statement (1) implies the figure is a parallelogram. In a parallelogram, opposite
angles are equal and the sum of all four angles must be 360 deg. Thus, if one of
the angles in a parallelogram is 90 deg, all of the angles are right angles and the
parallelogram is a rectangle.
#45
@gmat1.dll 145 100 100
4
~
What is the length of line segment AB? All lines that meet are perpendicular. AJ
JI, IH, BC, FE, GF and DC are each equal to x. HG and DE are each equal to y.
~
(1) y = 4
(2) x = 2
%
Since AB = JI + HG + FE + DC, if we knew the lengths of JI, HG, FE and DC we could
find the length of AB. So we need to know both what x is and what y is. Since DE
= IH + GF, y = 2x. Thus, either statement (1) alone or (2) alone is sufficient.
Marking sides that are equal to each other with either one slash or two slashes
will help you to see the relationships.
#46
@gmat1.dll 146 100 100
2
~
Is the angle z larger than 60 deg?
~
(1) x + y is greater than z.
(2) x is greater than y which is greater than z.
%
Statement (1) alone is not sufficient. if x = 70 deg, y = 25 deg and z = 85 deg,
Statement (1) would be true, however, if x = 90 deg, y = 45 deg, and z = 45 deg,
statement (1) would also be true. Statement (2) alone is sufficient. Since we
know that the sum of the angles in a triangle is 180 deg, the smallest angle is
always less than or equal to 60 deg. Notice that statement (2) gives enough
information to answer the question.
%
#47
@gmat1.dll 147 100 100
1
~
Will the circle with centre O fit inside the square PQRS?
~
(1) The diameter of the circle is less than a side of the square.
(2) The area of the circle is less than the area of the square.
%
Statement (1) alone is sufficient. Statement (2) is not sufficient.
#48
@gmat1.dll 148 150 100
3
~
Is RS parallel to PQ?
~
(1) RS = SP
(2) RT = TQ
%
Statement (1) alone is not sufficient since if RS equals SP, T can be any point on
the line RQ. By the same reasoning, since S could be any point on PR if statement
(2) is true, (2) alone is not sufficient. However, if both (1) and (2) are true,
then triangles RST and RPQ are similar, with the corresponding angles RTS and RQP
equal. Thus the transversal RQ has equal corresponding angles with the lines ST
and PQ, so ST is parallel to PQ. Thus (1) and (2) together are sufficient.
%
#49
@gmat1.dll 149 150 100
3
~
What is the value of x? AD = DC
~
(1) y = 30
(2) AB = BC
%
Statement (2) alone implies x = y since equal sides have equal angles in a
triangle. Since there is no information about y, statement (2) alone is
insufficient. Statement (1) alone is insufficient since there is no relation
between x and y without statement (2). Statements (1) and (2) together are
sufficient.
%
#50
@gmat1.dll 150 150 100
3
~
What is the area of rectangle PQRS?
~
(1) PR = 5
(2) PQ = 4
%
Since the figure is a rectangle, PQR is a right triangle. Therefore, using
statements (1) and (2) you can find QR, which will enable you to compute the area.
Statement (2) alone is not sufficient, since QR could be any value without
contradicting statement (2). Statement (1) alone is not sufficient. If PQ = 4 and
QR = 3, then PR = 5, and the area is 12. However, if PQ = 1 and QR = sq rt 24,
then PR = 5, but the area is sq rt 24 which is not 12.
%
#51
@gmat1.dll 151 150 100
1
~
PQRS is a square. What is the area of the triangle PQT?
~
(1) PQ = 10
(2) RT = ST
%
Since PQRS is a square, QR is an altitude of the triangle PQT and PQ is a base of
the triangle PQT. So if we know PQ ( which equals QR), we can determine the area.
Thus statement (1) alone is sufficient. Statement (2) alone is not sufficient
since it does not give any information about the length of any line segment.
%
#52
@gmat1.dll 152 150 100
1
~
PQR is a triangle inscribed in circle PORQ. Is PR a diameter of the circle PORQ?
~
(1) Angle PQR is a right angle.
(2) The length of PQ is 3/4 the length of QR.
%
Statement (1) alone is sufficient. If angle PQR is a right angle, then POR is a
semicircle. Therefore, PR is a diameter. Statement (2) alone is insufficient.
There are many ( an infinite number ) triangles we can inscribe in the circle such
that PQ = 3/4 QR. Not all of these will have PR as a diameter. Therefore,
statement (1) alone is sufficient, but statement (2) alone is not sufficient.
%
#53
@gmat1.dll 153 150 100
5
~
Is the triangle PQR congruent to the triangle MNO? a is equal to b.
~
(1) PQ is equal to MN.
(2) QR is equal to NO.
%
Statement (1) alone is not sufficient, since many noncongruent triangles can have a
side and an angle which are equal. By the same reasoning, statement (2) alone is
not sufficient. Statements (1) and (2) together are not sufficient. For two
triangles to be congruent , they must have two pairs of corresponding sides and the
included angles equal. For example, the following two triangle satisfy statements
(2) and (2) and a = b but they are not congruent. Therefore statements (1) and
(2) together are not sufficient.
%
#55
@gmat1.dll 155 150 100
1
~
What is the radius of the circle with centre O?
~
(1) The area of the circle is 25.
(2) The area of the circle divided by the diameter of the circle is equal to pi
times the diameter of the circle.
%
Since the area of a circle is equal to pixrxr and the radius is positive, statement
(1) alone is sufficient. Statement (2) is true for all circles, so it gives no
information about the radius of this particular circle.
%
#56
@gmat1.dll 156 150 100
4
~
Find the length of BC if z is 90 deg and AC = 6.
~
(1) AB = 6
(2) x = 45 deg
%
Since z is a right angle, (BC)(BC) = (AC)(AC) + (AB)(AB), so (BC)(BC) = (6)(6) +
(AB)(AB), and BC will be positive square root of 36 + (AB)(AB). Thus if you can
find the length of AC the problem is solved. Statement (1) says AB = 6, thus (BC)
(BC) = 36 + 36, so BC = 6 sq rt 2. Thus (1) alone is sufficient. Statement (2)
says x = 45 deg but since the sum of the angles in a triangle is 180 deg and z is
90 deg than y = 45 deg. So x and y are equal angles and that means the sides
opposite x and opposite y must be equal or AC = AB. Thus AB = 6 and BC = 6 sq rt
2. so (2) alone is also sufficient.
%
#57
@gmat1.dll 157 150 100
1
~
PR is a diameter of the circle. PRS is a straight line. What is the value of x?
~
(1) PQ = QR
(2) x = 2y
%
Since PR is a diameter, angle PQR is inscribed in a semicircle and is therefore a
right angle. Statement (1) alone is sufficient since it implies the two other
angles in the triangle must be equal. Since the sum of the angles of a triangle is
180 deg, we can deduce that x = 45. Statement (2) alone is not sufficient. There
is no information about the angle PQS, so statement (2) can not be used to find
the angles of triangle PQS.
%
#58
@gmat1.dll 158 150 100
1
~
PQ is perpendicular to RO. Is P or Q closer to R?
~
(1) OP is less than OQ.
(2) PQRS is not a parallelogram.
%
Statement (1) alone is sufficient. Draw the lines PR and QR, then POR and QOR are
right triangles, since PQ is perpendicular to RO. By the Pythagorean theorem, (PR)
(PR) = (PO)(PO) + (RO)(RO) and (QR)(QR) = (OQ)(OQ) + ((RO)(RO), so if PO is less
than OQ, then PR is less than QR. Statement (2) alone is not sufficient. There is
no restriction on where the point S is.
%
#59
@gmat1.dll 159 150 100
3
~
Does b = c?
~
(1) d = a
(2) PQRS is a parallelogram.
%
Statement (1) alone is not sufficient. By moving the point Q along the original
side QR, we can have either b = c or b not equal to c and still have d = a.
Statement (2) alone is not sufficient. It implies that b = d and c = a, but gives
no information to compare b and c. Statements (1) and (2) together, however,
yield b = c.
%
#60
@gmat1.dll 160 150 100
3
~
Does b = a?
~
(1) QS is perpendicular to PR.
(2) PQ is equal to QR.
%
Statement (2) alone is not sufficient, since S can be any point on the line PR if
we assume only statement (2). Statement (1) alone is not sufficient. Depending on
the position of point R, a and b can be equal or unequal. For example, in both of
the following triangles QS is perpendicular to PR. If statements (1) and (2) are
both true, then x = y. The triangles PQS and QSR are both right triangles with
two pairs of corresponding sides equal, the triangles are therefore congruent and a
= b.
%
#61
@gmat1.dll 161 150 100
5
~
Is PQRS a square?
~
(1) QR is perpendicular to PS.
(2) QT = TR.
%
Statements (1) and (2) together are not sufficient, since the points P and S can be
moved and statements (1) and (2) will still be satisfied.
%
#62
@gmat1.dll 162 150 100
1
~
Is ST parallel to QR ? SQ = PS.
~
(1) PT = TR
(2) SQ = TR
%
(1) alone is sufficient since the line connecting the midpoints of 2 sides of a
triangle is parallel to the third side. (2) alone is insufficient. In an isosceles
triangle statement (2) would imply that TS is parallel to QR, but in a non-
isosceles triangle, (2) would TS and QR are not parallel.
%
#63
5
~
Did Bill go the beach yesterday?
~
(1) If Bill goes to the beach, he will be sunburned the next day.
(2) Bill is sunburned today.
%
Obviously, neither statement alone is sufficient. Bill could have gotten sunburned
at the beach, but he might have gotten sunburned somewhere else. Therefore (1) and
(2) together are not sufficient.
%
#64
1
~
Is Y a number greater than zero?
~
(1) YxY + 1 = 0
(2) YxY -1 = 0.
%
Statement (1) means YxY = -1, but there is only one possible (real) solution to
this Y = -1. Thus Y is not greater than zero, which answers the question.
Statement (2) means YxY = 1, but there are two possible solutions to this equation,
Y = 1, Y = -1. Thus using (2) we cannot deduce whether Y is positive or negative.
And (1) alone is sufficient.
%
#65
5
~
An industrial plant produces containers. In 1981 the number of containers produced
by the plant was twice the number produced in 1980. How many containers were
produced altogether in the years 1980,1981,1982?
~
(1) In 1982 the number of containers produced was 3 times the number produced in
1980.
(2) In 1983 the number of containers produced was one half the total produced in
the years 1980,1981, and 1982.
%
T, the total produced in the three years, is the sum of P0 + P1 + P2, where P0 is
the number produced in 1980, P1 the number produced in 1981, and P2 the number
produced in 1982. You are given that P1 = 2P0. Thus T = P0+P1+P2 = 3P0+P2. So we
must find out P0 and P2 to answer the question. Statement (1) says P2 = 3P0 thus
by using (1) if we can find the value of P0 we can find T. But (1) gives us no
further information about P0. Statement (2) says T equals the number produced in
1983, but it does not say what this number is. Since there are no relations given
between production in 1983 and production in the individual years 1980,1981,or 1982
you cannot use (2) to find out what P0 is. Thus (1) and (2) together are not
sufficient.
%
#66
5
~
Is ab greater than 1? a and b are both positive?
~
(1) b is greater than 1
(2) a is less than 1.
%
Statements (1) and (2) together are not sufficient. If a = 3 and b = +, than ab is
greater than 1, but if a = 3/2 and b = +, then ab is less than 1.
%
#67
4
~
A piece of iron rod 5 feet long is cut into three smaller pieces. How long is the
longest of the three pieces?
~
(1) One piece is 2 feet 7 inches long.
(2) One piece is 7 inches longer than another piece and the remaining piece is 5
inches long.
%
Statement (1) alone is sufficient. 2 feet 7 inches is more than of 5 feet, so the
piece which is 2 feet 7 inches long must be longer than the other two pieces put
together. Statement (2) alone is sufficient. Since one piece is 5 inches long,
the sum of the remaining two pieces is 4 feet, 7 inches. Since one piece is 7
inches longer than the other, L + (L +7 in.) = 4 ft. 7 in., where L is the length
of the smaller of the two remaining pieces. Solving the equations yields L + 7 in.
as the length of the longest piece.
%
#68
2
~
What is the value of b?
~
(1) a + b = 8
(2) bxb - 2b + 1 = 0
%
Statement (2) alone is sufficient, bxb - 2b + 1 equals (b-1)(b-1), and the only
solution of (b-1)(b-1) =0 is b =1. Statement (1) alone is not sufficient. a + b =
8 implies b = 8 - a, but there are no data given about the value of a.
%
#69
2
~
What is the value of a+b?
~
(1) a -b = 8
(2) 3a + 3b = 8
%
Statement (2) alone is sufficient, since 3a + 3b is 3(a+b) , then a+b = 8 / 3.
Statement (1) alone is not sufficient, since you need another equation besides a -
b = 8 to find the values of a and b.
%
#70
5
~
How far is it from X to Y ?
~
(1) It is 10 miles from X to Z
(2) It is 20 miles from Z to Y
%
Statements (1) and (2) together are not sufficient, because there is no information
about the location of Z relative to the locations of X and Y.
%
#71
2
~
Is m an even integer? Assume a and b are integers.
~
(1) m = ( a+ b)(a+b)
(2) m = 2a + 10b
%
An even integer is an integer divisible by 2. Since 2a + 10b is 2 times (a+5b)
using (2) lets you deduce that m is even. (1) by itself is not sufficient. If a
were 2 and b were 3, (a+b)(a+b) would be 25 which is not even, but by choosing a
to be 2 and b to be 4, (a +b)(a+b) is 36 which is even.
%
#72
3
~
What was the price of a dozen apples during the 15th week of the year 1996?
~
(1) During first week of 1996 the price of a dozen apples was $ 75/-.
(2) The price of a dozen apples rose $ 1/- a week every week during the first four
months of 1996.
%
You need (1) to know what the price was at the beginning of 1996. Using (2) you
could then compute the price during the fifteenth week. Either statement alone is
insufficient. You should not actually compute the price since it would only waste
time.
%
#73
5
~
Is a > b ?
~
(1) a/b = 5/4
(2) axa > bxb
%
If you answered incorrectly you probably assumed that a and b were positive. If a
= 5 and b = 4, then (1) and (2) are both true and a > b. However, if a = -5 and b
= -4, (1) and (2) are both true but a <b.
%
#74
1
~
Does every bird fly?
~
(1) Peahens do not fly
(2) Lions do not fly
%
(1) alone is sufficient since Peahens are birds. (2) alone is not sufficient since
Lions are not birds.
%
#75
5
~
Find a+ 2b?
~
(1) a - b = 12
(2) 3a - 3b = 36
%
Since the equation in (2) has exactly the same solutions as the equation in (1),
3(a-b) = 3a - 3b and 3(12) = 36, you can't determine a and b even by using both (1)
and (2). If a = 12 and b =0, then (1) and (2) are true and a+2b = 12, but if a =
6 and b = -6, (1) and (2) are again true but a+2b = -6.
%
#76
1
~
What is the value of 2(x-y)?
~
(1) 5x - 5y = 25
(2) 3x + 2y = 10
%
Statement (1) alone is sufficient, since 5x - 5y is 5(x-y). Therefore x-y = 25/5 =
5, 2(x-y) = 2(5) = 10. Statement (2) alone is not sufficient, since you need
another equation besides 3x + 2y = 4 to find the values of x and y.
%
#77
5
~
Is xy greater than 1? x and y are both positive.
~
(1) x is less than 1
(2) y is greater than 1
%
Statements (1) and (2) together are not sufficient. If x = 1/2 and y = 3, then xy
is greater than 1, but if x = 1/5 and y = 10, then xy is greater than 1, but if x =
1/5 and y = 1/3, then xy is less than 1.
%
#78
5
~
How far is it from M to N?
~
(1) It is 30 miles from A to C
(2) It is 25 miles from M to A
%
Statements (1) and (2) together are not sufficient, because there is no information
about the location of A and C relative to the locations of M and N,
%
#79
1
~
Is x an even integer? Assume n and p are integers.
~
(1) x =4n +40p
(2) x = ( n+p)(n+p)
%
An even integer is an integer divisible by 2. Since 4n + 40p is 4 times(n+10p)
using (1) lets you deduce that x is even. (2) by itself is not sufficient. If n
were 2 and p were 3, (n+p)(n+p) would be 25 which is not even, but by choosing n to
be 2 and p to be 4, (n+p)(n+p) is 36 which is even.
%
#80
5
~
Did the price of lumber rise by more than 20% last year?
~
(1) Lumber exports increased by 30%
(2) The amount of timber cut decreased by 10%
%
Both statements give facts that might explain why the price of lumber rose.
However, even using both statements you can't deduce what happened to the price of
lumber.
%
#81
5
~
Is m<n
~
(1) mxm < nxn
(2) m/n = 8/10
%
If m = 8 and n = 10, then (1) and (2) are both true and m < n. However, if m =
-8 and n = -10, (1) and (2) are both true but m > n.
%
#82
2
~
Find the value of the expression XxXxY -(XxX/Y)
~
(1) X = 2
(2) Y = 1
%
If (1) is true, then XxXxY -(XxX/Y) is equal to 4Y-(4/Y), but the value is Y is
needed to find the value of the expression. Therefore,(1) alone is not
sufficient. If statement (2) alone is true, then XxXxY -(XxX/Y) is equal to XxX
- XxX, which is equal to 0. Therefore (2) alone is sufficient. So the correct
choice is B.
%
#83
5
~
If a is a two-digit number, what is the last digit x of a?
~
(1) The number 3a is a three-digit number whose last digit of is x
(2) The digit x is less than 7.
%
If (1) is true, then since a = yx, 3a = 3(10y + x) = 30y + 3x. Now, because y is
multiplied by 10 in the expression for 3a, the final digit of 3a must be the final
digit of 3x. Since x is a digit, 0<= x <= 9, which implies o <= 3x <= 27. So for
the last digit of 3x to be equal to x, 3x must equal x or 10 + x = 3x, then 10 =
2x or x = 5, If 20 + x = 3x, then 20 = 2a or a = 10, but since 10 is not a digit
this is not possible. So if (1) is true, then x is 0 or 5, and (1) alone is not
sufficient. Now since 26 and 25 are both two-digit numbers whose last digit are
less than 7, (2) alone is not sufficient. So statements (1) and (2) together are
not sufficient.
%
#84
2
~
Is the number A/3 an odd integer? (You may assume that A/3 is an integer)
~
(1) A = 3B where B is an integer.
(2) A = 6C + 3 where C is an integer.
%
(1) alone is not sufficient since then A/3 = (3B)/3 = B. Now if B = 1, then A/3 =
1, which is odd, but if B = 2, then A /3 = 2, which is even. (2) alone is
sufficient since then A/3 = (6C +3)/3 = 2C + 1, which is always odd since C is an
integer
%
#85
2
~
Is the number A/3 an odd integer? (You may assume that A/3 is an integer)
~
(1) A = 3B where B is an integer.
(2) A = 6C + 3 where C is an integer.
%
(1) alone is not sufficient since then A/3 = (3B)/3 = B. Now if B = 1, then A/3 =
1, which is odd, but if B = 2, then A /3 = 2, which is even. (2) alone is
sufficient since then A/3 = (6C +3)/3 = 2C + 1, which is always odd since C is an
integer
%
#86
2
~
Is a greater than zero?
~
(1) a + bxb = 16
(2) axaxa - 8 = 0
%
axaxa - 8 = 0 has only a = 2 as a real solution. And 2 is greater than zero, so
statement (2) alone is sufficient. (1) alone is not sufficient since you need the
value for b to solve a.
%
#88
2
~
Is the integer m divisible by 3?
~
(1) The last digit in m is 3.
(2) m + 5 is divisible by 6.
%
Statement (1) alone is not sufficient. If m is 33, then (1) is true , and m is
divisible by 3, but if m is 13, then (1) is true, but m is not divisible by 3.
(2) is sufficient. According to (2) there must be an integer n such that m + 5 =
6n, so m is 6n - 5. But this means that m divided by 3 will be 2n + 5 / 3, so m
is not divisible by 3. So (2) is sufficient.
%
#89
4
~
How many degree Celsius is 100degree Fahrenheit?
~
(1) degree Celsius = 5/9 (degrees Fahrenheit - 32)
(2) degree Fahrenheit = 9/5(degrees Celsius) + 32.
%
Statement (1) alone is sufficient. Just use 100 for degrees Fahrenheit in the
formula. Statement (2) alone is also sufficient, since the formula in statement (2)
can be solved to give the formula of statement (1) which we know is sufficient.
%
#90
5
~
Is x > y ?
~
(1) (x + y)(x+y) > 0
(2) x is positive.
%
Because x = 2, y = 1 and x = 1, y = 2 both satisfy statements (1) and (2), the
statements together are not sufficient.
%
#91
5
~
What is a+ b + c?
~
(1) a + b = 3
(2) a + c = 2
%
If we use statement (1), we have a + b+ c = 3 + c, but we have no information about
c, so (1) alone is insufficient. If we use (2) alone, we have a+b+c = b +2, but
since we have no information about b, (2) alone is insufficient. If we use both
(1) and (2), we obtain a+b+c = y+ 2= 3+z. We can also add (1) and (2) to obtain
2a + b+c = 5, but we can't find the value of a+b+c without more information. So
the answer is E.
%
#92
2
~
Is a number divisible by 9?
~
(1) The number is divisible by 3
(2) The number is divisible by 36
%
Statement (1) alone is not sufficient, since 12 is divisible by 3 but 12 is not
divisible by 9. Statement (2) alone is sufficient, since if a number is divisible
by 36 then, because 36 = 9 x 4, the number must be divisible by 9.
%
#93
1
~
Is the integer x odd or even.
~
(1) xXx is odd.
(2) 2x is even.
%
The square of an even integer is always even. So if xXx is odd, x can't be even.
Therefore, x is odd and (1) alone is sufficient. Statement (2) alone is not
sufficient, since 2x is even for every integer x.
%
#94
3
~
Is m positive ?
~
(1) mxm + 3m - 4 = 0
(2) m > -2
%
Statement (1) alone is not sufficient. Factoring the expression in (1) gives m is
equal to -4 or 1. Statement (2) alone is not sufficient, since there are positive
numbers greater than -2 and negative numbers greater than -2 . Statement (1) and
(2) together are sufficient, since the only possible value is 1.
%
#95
5
~
What was Mr. Lara's income in 1995?
~
(1) His total income for 1993, 1994, and 1995 was $199,000.
(2) He made 20% more in 1994 than he did in 1993.
%
Using statement (1) we can find the income for 1995 if we know the income for 1993
and 1994, but (1) gives no information about the income for 1993 and 1994. If we
also use (2) we can get the income in 1994 if we know the income for 1993, but we
still can't determine the income for 1993. Therefore, both together are not
sufficient.
%
#96
5
~
Is a greater than b.
~
(1) 3a = 2m.
(2) m = bxb.
%
Since statement (1) describes only a and statement (2) describes only b, both are
needed to get an answer. Using statement (2), statement (1) becomes 3a = 2m =
2bxb, so a = 2bxb / 3. However, this is not sufficient, since if b = -1 then a=
2/3 and a is greater than b, but if b = 1 then again a = 2/3 but now a is less
than b.
%
#97
1
~
What was Mr. John's combined income for the years 1990-1995? In 1990 he made
$20,000.
~
(1) His average yearly income for the years 1990-1995 was $22,000.
(2) In 1995, his income was $30,000.
%
Statement (1) alone is sufficient. The average is the combined income for 1990-
1995 divided by 6. Therefore, the combined income is 6 times the average yearly
income. Statement (2) alone is not sufficient since there is no information about
his income for the years 1991-1994.
%
#98
2
~
What is the value of a-b?
~
(1) a + 2b = 6
(2) a = b
%
Statement (2) alone is sufficient, since a = b implies a-b =0. Statement (1)
alone is not sufficient. An infinite number of pairs satisfy statement(1), for
example, a = 2, b = 2, for which a-b = 0, or a = 4, b = 1, for which a-b = 3.
%
#99
5
~
The number of eligible voters is 200,000. How many eligible voters voted?
~
(1) 60% of the eligible men voted.
(2) 65% of the eligible women voted.
%
Since there is no information on how many of the eligible voters are men or how
many are women, statements (1) and (2) together are not sufficient.
%
#100
1
~
How much was the original cost of a car which sold for $3,100?
~
(1) The car was sold for a discount of 20% from its original cost.
(2) The sales tax was $150.
%
Statement (1) is sufficient, since it means 90% of the original cost is $3,100.
Thus, we can solve the equation for the original cost. Statement (2) alone is not
sufficient, since it gives no information about the cost.
%
#
5
~
Are two triangles congruent ?
~
1. Both triangles are right triangles.
2. Both triangles have the same perimeter.
%
The correct answer is
(E). Statements (1) and (2) TOGETHER are NOT sufficient
to answer the question asked, and additional data
specific to the problem are needed.
%
#
2
~
Is a > 0 ?
~
1. (a*a*a*a) - 16 = 0
2. a*a*a - 8 = 0
%
The correct answer is
(B). Statement (2) ALONE is sufficient, but statement (1) alone
is not sufficient to answer the question asked;
%
#
4
~
If both conveyer belt A and conveyer belt B are used, they can fill a hopper with
coal in one hour. How long will it take for the conveyer belt A to fill the hopper
without conveyer belt B.
~
1. Conveyer belt A moves twice as much coal as conveyer belt B.
2. Conveyer belt B would take 3 hours to fill the hopper without belt A.
%
The correct answer is
(D). EACH statement ALONE is sufficient
to answer the question asked;
%
#
3
~
Is x larger than 1 ?
~
1. x is larger than 0.
2. x*x - 4 > 0
%
The correct answer is
(C). BOTH statements (1) and (2) TOGETHER are
sufficient to answer the question asked,
but NEITHER statement ALONE is sufficient;
%
#
4
~
What is the area of a circle ?
~
1. The circumference is 12pi
2. The diameter is 12
%
The correct answer is
(D). EACH statement ALONE is sufficient
to answer the question asked;
%
#
1
~
What is the value of x ?
~
3x + 12 = 36
5x + 3y = 16
%
The correct answer is
(A). Statement (1) ALONE is sufficient, but
statement (2) alone is not sufficient
to answer the question asked;
%
#
3
~
What is the average height of Arnold, Bob and Charles?
~
1. Arnold is 6 inches shorter than Bob and Charles is 6 inches longer than Bob.
2. Bob is 6 feet tall.
%
The correct answer is
(C). BOTH statements (1) and (2) TOGETHER are
sufficient to answer the question asked,
but NEITHER statement ALONE is sufficient;
%
#
5
~
What is the average height of Arnold, Bob and Charles?
~
1. Arnold is 5 inches shorter than Bob and Charles is 5 inches longer than Bob.
2. Bob is 24 years old.
%
The correct answer is
(E). Statements (1) and (2) TOGETHER are NOT sufficient
to answer the question asked, and additional data
specific to the problem are needed.
%
#
1
~
What is the value of a ?
~
9a + 12 = 50
5a + 3b = 16
%
The correct answer is
(A). Statement (1) ALONE is sufficient, but
statement (2) alone is not sufficient
to answer the question asked;
%
#
3
~
What is the value of y ?
~
3x + 12 = 36
x + 3y = 16
%
The correct answer is
(C). BOTH statements (1) and (2) TOGETHER are
sufficient to answer the question asked,
but NEITHER statement ALONE is sufficient;
%
#
3
~
What is the value of y ?
~
3x + 6 = 36
5x + 3y = 80
%
The correct answer is
(C). BOTH statements (1) and (2) TOGETHER are
sufficient to answer the question asked,
but NEITHER statement ALONE is sufficient;
%
#
3
~
What is the value of x + y ?
~
3x + 12 = 36
3y + 24 = 36
%
The correct answer is
(C). BOTH statements (1) and (2) TOGETHER are
sufficient to answer the question asked,
but NEITHER statement ALONE is sufficient;
%
#
4
~
What is the value of x + y ?
~
3x + 3y = 36
5x + 5y = 60
%
The correct answer is
(D). EACH statement ALONE is sufficient
to answer the question asked;
%
#
3
~
What is the individual heights of Arnold, Bob and Charles?
~
1. Arnold is 6 inches shorter than Bob and Charles is 6 inches longer than Bob.
2. Bob is 6 feet tall.
%
The correct answer is
(C). BOTH statements (1) and (2) TOGETHER are
sufficient to answer the question asked,
but NEITHER statement ALONE is sufficient;
%
#
3
~
What is the average height of Arnold, Bob and Charles?
~
1. Arnold is 6 inches shorter than Bob and Charles is 6 inches longer than Bob.
2. Arnold's height is 5ft.
%
The correct answer is
(C). BOTH statements (1) and (2) TOGETHER are
sufficient to answer the question asked,
but NEITHER statement ALONE is sufficient;
%
#
1
~
What is the average height of Arnold, Bob and Charles?
~
1. Arnold is 6 inches shorter than Bob and Charles is 6 inches longer than Bob and
Arnold's height is 5ft.
2. Bob is 6 feet tall.
%
The correct answer is
(A). Statement (1) ALONE is sufficient, but
statement (2) alone is not sufficient
to answer the question asked;
%
#
5
~
What is the runner's average speed in running around a field ?
~
1. One lap around the field took 1 minute.
2. He ran 5 seconds faster than his previous best average.
%
The correct answer is
(E). Statements (1) and (2) TOGETHER are NOT sufficient
to answer the question asked, and additional data
specific to the problem are needed.
%
#
3
~
What is the runner's average speed in running around a circular field ?
~
1. One lap around the field took 6 minute.
2. The circumference of the field is 1000 meters.
%
The correct answer is
(C). BOTH statements (1) and (2) TOGETHER are
sufficient to answer the question asked,
but NEITHER statement ALONE is sufficient;
%
#
3
~
What is the runner's average speed in running around a circular field ?
~
1. One lap around the field took 6 minutes.
2. The radius of the circular field is 50 meters.
%
The correct answer is
(C). BOTH statements (1) and (2) TOGETHER are
sufficient to answer the question asked,
but NEITHER statement ALONE is sufficient;
%
#
3
~
What is the runner's average speed in running around a circular field ?
~
1. One lap around the field took 6 minutes.
2. The diameter of the circular field is 100 meters.
%
The correct answer is
(C). BOTH statements (1) and (2) TOGETHER are
sufficient to answer the question asked,
but NEITHER statement ALONE is sufficient;
%
#
1
~
What is the runner's average speed in running around a circular field ?
~
1. One lap around the field took 6 minutes and the circumference of the field is
1000 meters.
2. He ran 5 seconds faster than his previous best average.
%
The correct answer is
(A). Statement (1) ALONE is sufficient, but
statement (2) alone is not sufficient
to answer the question asked;
%
#
3
~
Who is the tallest of the four men ?
~
1. John is shorter than Steven.
2. Arnold and Steven are shorter than Robert.
%
The correct answer is
(C). BOTH statements (1) and (2) TOGETHER are
sufficient to answer the question asked,
but NEITHER statement ALONE is sufficient;
%
#
5
~
What is the height of Arnold ?
~
1. John is shorter than Steven.
2. Arnold and Steven are shorter than Robert.
%
The correct answer is
(E). Statements (1) and (2) TOGETHER are NOT sufficient
to answer the question asked, and additional data
specific to the problem are needed.
%
#
5
~
Who is the shortest of the four men ?
~
1. John is shorter than Steven.
2. Arnold and Steven are shorter than Robert.
%
The correct answer is
(E). Statements (1) and (2) TOGETHER are NOT sufficient
to answer the question asked, and additional data
specific to the problem are needed.
%
#
3
~
What was the percentage increase in Mr Alex's spending ?
~
1. He is now spending $ 100 more than what he spent last month.
2. He is now spending $ 500 per month.
%
The correct answer is
(C). BOTH statements (1) and (2) TOGETHER are
sufficient to answer the question asked,
but NEITHER statement ALONE is sufficient;
%
#
1
~
What was the percentage increase in Mr Alex's spending ?
~
1. He is now spending $ 100 more than what he spent last month and last month he
spent $ 500.
2. He is now earning $ 1500 per month.
%
The correct answer is
(A). Statement (1) ALONE is sufficient, but
statement (2) alone is not sufficient
to answer the question asked;
%
#
5
~
What was the percentage increase in Mr Alex's spending ?
~
1. He is now earning $ 3000 per month.
2. He is paying $ 300 as income tax every month.
%
The correct answer is
(E). Statements (1) and (2) TOGETHER are NOT sufficient
to answer the question asked, and additional data
specific to the problem are needed.
%
#
5
~
Is A = C ?
~
1. A is not equal to B.
2. B is not equal to C.
%
The correct answer is
(E). Statements (1) and (2) TOGETHER are NOT sufficient
to answer the question asked, and additional data
specific to the problem are needed.
%
#
3
~
Is x = z ?
~
1. x is equal to y.
2. y is equal to z.
%
The correct answer is
(C). BOTH statements (1) and (2) TOGETHER are
sufficient to answer the question asked,
but NEITHER statement ALONE is sufficient;
%
#
5
~
Is x = z ?
~
1. x = 13
2. y = 11 + z
%
The correct answer is
(E). Statements (1) and (2) TOGETHER are NOT sufficient
to answer the question asked, and additional data
specific to the problem are needed.
%
#
2
~
What is the value of a/b ?
~
1. ab=14
2. b=6a
%
The correct answer is
(B). Statement (2) ALONE is sufficient, but statement (1) alone
is not sufficient to answer the question asked;
%
#
4
~
What is the value of a/b ?
~
1. a=b/6
2. b=6a
%
The correct answer is
(D). EACH statement ALONE is sufficient
to answer the question asked;
%
#
3
~
How many brothers does Daniel have ?
~
1. His parents have seven surviving children.
2. He has twice as many sisters as brothers.
%
The correct answer is
(C). BOTH statements (1) and (2) TOGETHER are
sufficient to answer the question asked,
but NEITHER statement ALONE is sufficient;
%
#
3
~
How many sisters does Daniel have ?
~
1. His parents have seven surviving children.
2. He has twice as many sisters as brothers.
%
The correct answer is
(C). BOTH statements (1) and (2) TOGETHER are
sufficient to answer the question asked,
but NEITHER statement ALONE is sufficient;
%
#
5
~
How many brothers does Daniel have ?
~
1. His parents were married 25 years ago.
2. He has twice as many sisters as brothers.
%
The correct answer is
(E). Statements (1) and (2) TOGETHER are NOT sufficient
to answer the question asked, and additional data
specific to the problem are needed.
%
#
1
~
How many brothers does Daniel have ?
~
1. His parents have seven surviving children among whom are four girls.
2. He has twice as many sisters as brothers.
%
The correct answer is
(A). Statement (1) ALONE is sufficient, but
statement (2) alone is not sufficient
to answer the question asked;
%
#
4
~
How many brothers does Daniel have ?
~
1. His parents have seven surviving children among whom are four girls.
2. He has twice as many sisters as brothers and Daniel's parents have seven
surviving children.
%
The correct answer is
(D). EACH statement ALONE is sufficient
to answer the question asked;
%
#
5
~
Which is more expensive, an apple or an orange ?
~
1. A dozen apples costs $5.
2. Oranges cost $2 a pound.
%
The correct answer is
(E). Statements (1) and (2) TOGETHER are NOT sufficient
to answer the question asked, and additional data
specific to the problem are needed.
%
#
3
~
Which is more expensive, an apple or a mango ?
~
1. A dozen apples costs $5.
2. A dozen mangoes cost $2.
%
The correct answer is
(C). BOTH statements (1) and (2) TOGETHER are
sufficient to answer the question asked,
but NEITHER statement ALONE is sufficient;
%
#
5
~
What is the cost of two apples and two oranges ?
~
1. A dozen apples costs $5.
2. Oranges cost $5 a pound.
%
The correct answer is
(E). Statements (1) and (2) TOGETHER are NOT sufficient
to answer the question asked, and additional data
specific to the problem are needed.
%
#
1
~
Is x > y ?
~
1. (x-y) > 0
2. x*x < y*y
%
The correct answer is
(A). Statement (1) ALONE is sufficient, but
statement (2) alone is not sufficient
to answer the question asked;
%
#
4
~
Is x > y ?
~
1. (x-y) > 0
2. 2x > 2y
%
The correct answer is
(D). EACH statement ALONE is sufficient
to answer the question asked;
%
#
5
~
What was the interest rate in an savings account ?
~
1. The account holder collected $ 50 in simple interest over several months.
2. There was $ 2000 in the account initially.
%
The correct answer is
(E). Statements (1) and (2) TOGETHER are NOT sufficient
to answer the question asked, and additional data
specific to the problem are needed.
%
#
3
~
What was the interest rate in an savings account ?
~
1. The account holder collected $ 50 in simple interest over twelve months.
2. There was $ 2000 in the account initially.
%
The correct answer is
(C). BOTH statements (1) and (2) TOGETHER are
sufficient to answer the question asked,
but NEITHER statement ALONE is sufficient;
%
#
3
~
Which is smallest of the three numbers that average 7 ?
~
1. One of the numbers is 6.
2. One of the numbers is 8.
%
The correct answer is
(C). BOTH statements (1) and (2) TOGETHER are
sufficient to answer the question asked,
but NEITHER statement ALONE is sufficient;
%
#
3
~
Which is largest of the three numbers that average 7 ?
~
1. One of the numbers is 6.
2. One of the numbers is 8.
%
The correct answer is
(C). BOTH statements (1) and (2) TOGETHER are
sufficient to answer the question asked,
but NEITHER statement ALONE is sufficient;
%
#
4
~
Which is smallest of the three numbers that average 7 ?
~
1. Two of the three numbers are 6 and 7.
2. Two of the three numbers are 7 and 8.
%
The correct answer is
(D). EACH statement ALONE is sufficient
to answer the question asked;
%
#
1
~
Which is smallest of the three numbers that average 7 ?
~
1. Two of the three numbers are 6 and 7.
2. One of the three numbers is 8.
%
The correct answer is
(A). Statement (1) ALONE is sufficient, but
statement (2) alone is not sufficient
to answer the question asked;
%
#
4
~
In the equation 6x - 4y - 20 = 0, what is the value of y ?
~
1. x=y
2. x=10
%
The correct answer is
(D). EACH statement ALONE is sufficient
to answer the question asked;
%
#
4
~
Given the equation 6x - 4y - 20 = 0, what is the value of x + y ?
~
1. x=y
2. x=10
%
The correct answer is
(D). EACH statement ALONE is sufficient
to answer the question asked;
%
#
4
~
What is the area of a circular field ?
~
1. The circumference is 314 feet.
2. The diameter is 100 feet.
%
The correct answer is
(D). EACH statement ALONE is sufficient
to answer the question asked;
%
#
4
~
What is the area of a circular field ?
~
1. The radius is 50 feet.
2. The diameter is 100 feet.
%
The correct answer is
(D). EACH statement ALONE is sufficient
to answer the question asked;
%
#
4
~
A hexagonal playing field is composed of six equilateral triangles, each of the
same size. If one player controls each triangle, how much area is controlled by all
six players ?
~
1. One edge of one triangular region equals 15 feet.
2. The perimeter of one triangular region equals 45 feet.
%
The correct answer is
(D). EACH statement ALONE is sufficient
to answer the question asked;
%
#
3
~
How many Mondays are in December of a particular year ?
~
1. There are 4 Tuesdays that December.
2. There are 4 Saturdays that December.
%
The correct answer is
(C). BOTH statements (1) and (2) TOGETHER are
sufficient to answer the question asked,
but NEITHER statement ALONE is sufficient;
%
#
4
~
How long is the diagonal through the centre of a particular cube ?
~
1. A diagonal across one face is 4.24 centimetres.
2. The surface area of the cube is 54 square centimetres.
%
The correct answer is
(D). EACH statement ALONE is sufficient
to answer the question asked;
%
#
5
~
Were more new cars or new jeeps purchased by the people of town X ?
~
One fourth of the population on town X purchased new cars.
One third of the population on town X purchased new jeeps.
%
The correct answer is
(E). Statements (1) and (2) TOGETHER are NOT sufficient
to answer the question asked, and additional data
specific to the problem are needed.
%
#
3
~
Is a greater than b ?
~
1. a = 1
2. b is a prime number
%
The correct answer is
(C). BOTH statements (1) and (2) TOGETHER are
sufficient to answer the question asked,
but NEITHER statement ALONE is sufficient;
%
#
3
~
How many rectangular tiles are required for the kitchen floor ?
~
1. Each tile is 48 square inches.
2. The kitchen measures 10 feet by 7 feet.
%
The correct answer is
(C). BOTH statements (1) and (2) TOGETHER are
sufficient to answer the question asked,
but NEITHER statement ALONE is sufficient;
%
#
3
~
How many squarish tiles are required for the kitchen floor which is a perfect
square ?
~
1. The tile has a length of 10 inches.
2. The kitchen measures 100 inches By 100 inches.
%
The correct answer is
(C). BOTH statements (1) and (2) TOGETHER are
sufficient to answer the question asked,
but NEITHER statement ALONE is sufficient;
%
#
5
~
How many rectangular tiles are required for the kitchen floor ?
~
1. The tile's length is 12 inches.
2. The tile's width is 10 inches.
%
The correct answer is
(E). Statements (1) and (2) TOGETHER are NOT sufficient
to answer the question asked, and additional data
specific to the problem are needed.
%
#
5
~
How many rectangular tiles are required for the kitchen floor ?
~
1. The tile's length is 10 inches.
2. The kitchen's length is 10 feet.
%
The correct answer is
(E). Statements (1) and (2) TOGETHER are NOT sufficient
to answer the question asked, and additional data
specific to the problem are needed.
%
#
4
~
How much rainfall does City A need this month to attain its long term average
rainfall of 35 inches per year ?
~
1. Over the last 11 months, the city A had an average rainfall of 3 inches per
month.
2. City A has accumulated 94.3% of its mean annual rainfall during the last 11
months.
%
The correct answer is
(D). EACH statement ALONE is sufficient
to answer the question asked;
%
#
1
~
How much rainfall does City A need this month to attain its long term average
rainfall of 35 inches per year ?
~
1. Over the last 11 months, the city A had an average rainfall of 3 inches per
month.
2. The average rainfall of City A in the past 11 months is more than the average
rainfall of city B.
%
The correct answer is
(A). Statement (1) ALONE is sufficient, but
statement (2) alone is not sufficient
to answer the question asked;
%
#
2
~
How much rainfall does City A need this month to attain its long term average
rainfall of 35 inches per year ?
~
1. Over the last 5 months, the city A had a rainfall of 15 inches.
2. City A has accumulated 94.3% of its mean annual rainfall during the last 11
months.
%
The correct answer is
(B). Statement (2) ALONE is sufficient, but statement (1) alone
is not sufficient to answer the question asked;
%
#
3
~
What is the range of a truck on one tank of gasoline ?
~
1. The tank holds 85 litres.
2. It consumes 7 litres of gasoline each 100 kilometres.
%
The correct answer is
(C). BOTH statements (1) and (2) TOGETHER are
sufficient to answer the question asked,
but NEITHER statement ALONE is sufficient;
%
#
5
~
What is the maximum speed of a truck ?
~
1. The tank holds 85 litres.
2. It consumes 7 litres of gasoline each 100 kilometres.
%
The correct answer is
(E). Statements (1) and (2) TOGETHER are NOT sufficient
to answer the question asked, and additional data
specific to the problem are needed.
%
#
3
~
What is the range of a sports car on exactly half tank of gasoline ?
~
1. The tank holds 45 litres.
2. It consumes 9 litres of gasoline each 100 kilometres.
%
The correct answer is
(C). BOTH statements (1) and (2) TOGETHER are
sufficient to answer the question asked,
but NEITHER statement ALONE is sufficient;
%
#
1
~
Five persons (A, B, C, D and E) sat next to each other around a circular table to
play cards. Did A sit next to B?
~
1. C sat next to D and E.
2. E sat next to A.
%
The correct answer is
(A). Statement (1) ALONE is sufficient, but
statement (2) alone is not sufficient
to answer the question asked;
%
#
5
~
Five persons (A, B, C, D and E) sat next to each other around a circular table to
play cards. Did A sit next to B?
~
1. C sat next to D.
2. E sat next to A.
%
The correct answer is
(E). Statements (1) and (2) TOGETHER are NOT sufficient
to answer the question asked, and additional data
specific to the problem are needed.
%
#
2
~
Five persons (A, B, C, D and E) sat next to each other around a circular table to
play cards. Did A sit next to B?
~
1. B sat next to E.
2. D sat between C and E.
%
The correct answer is
(B). Statement (2) ALONE is sufficient, but statement (1) alone
is not sufficient to answer the question asked;
%
#
5
~
What is M's age ?
~
1. In 2 years, M will be twice as old as B is now.
2. S's age is the average of M's and B's ages.
%
The correct answer is
(E). Statements (1) and (2) TOGETHER are NOT sufficient
to answer the question asked, and additional data
specific to the problem are needed.
%
#
3
~
What is M's age ?
~
1. In 2 years, M will be twice as old as B is now.
2. B is 8 years old.
%
The correct answer is
(C). BOTH statements (1) and (2) TOGETHER are
sufficient to answer the question asked,
but NEITHER statement ALONE is sufficient;
%
#
1
~
What is the value of (14x) / y ?
~
1. x = 2y
2. y = 5
%
The correct answer is
(A). Statement (1) ALONE is sufficient, but
statement (2) alone is not sufficient
to answer the question asked;
%
#
2
~
A pair of shoes was discounted to $ 75. What was the list price ?
~
1. There would be a 10% fee for exchange of shoes.
2. The sale was at "25% off".
%
The correct answer is
(B). Statement (2) ALONE is sufficient, but statement (1) alone
is not sufficient to answer the question asked;
%
#
4
~
A pair of shoes was discounted to $ 75. What was the list price ?
~
1. The purchaser paid $ 25 less than the list price.
2. The sale was at "25% off".
%
The correct answer is
(D). EACH statement ALONE is sufficient
to answer the question asked;
%
#
5
~
In the expression 3a - b < c, is b positive ?
~
1. a=3
2. c=17
%
The correct answer is
(E). Statements (1) and (2) TOGETHER are NOT sufficient
to answer the question asked, and additional data
specific to the problem are needed.
%
#
3
~
What is the value of a ?
~
1. 2a - 3b = 11
2. a - b = 5
%
The correct answer is
(C). BOTH statements (1) and (2) TOGETHER are
sufficient to answer the question asked,
but NEITHER statement ALONE is sufficient;
%
#
4
~
Is m, which is not equal to zero, a positive number ?
~
1. m = m*m
2. m*m = m*m*m
%
The correct answer is
(D). EACH statement ALONE is sufficient
to answer the question asked;
%
#
3
~
How long would it take S to clean the room ?
~
1. Working with B, the room is cleaned in 2 hours.
2. B can finish the cleaning working alone in 3.5 hours.
%
The correct answer is
(C). BOTH statements (1) and (2) TOGETHER are
sufficient to answer the question asked,
but NEITHER statement ALONE is sufficient;
%
#
4
~
Is a < 2?
~
1. a > a*a
2. (1/a) > 3
%
The correct answer is
(D). EACH statement ALONE is sufficient
to answer the question asked;
%
#
3
~
What is the individual price of three radios ?
~
1. The three radios have an average price of $200.
2. Two are identical and sell for $125 together.
%
The correct answer is
(C). BOTH statements (1) and (2) TOGETHER are
sufficient to answer the question asked,
but NEITHER statement ALONE is sufficient;
%
#
5
~
What is the individual price of three televisions ?
~
1. The three televisions have an average price of $200.
2. One of the television costs $125.
%
The correct answer is
(E). Statements (1) and (2) TOGETHER are NOT sufficient
to answer the question asked, and additional data
specific to the problem are needed.
%
#
5

~
How many black shoes were sold by shop X?
~
1. Three fourth's of the shoes sold were brown.
2. The shop X sold 1500 left shoes.
%
The correct answer is
(E). Statements (1) and (2) TOGETHER are NOT sufficient
to answer the question asked, and additional data
specific to the problem are needed.
%
#
3

~
A and his wife B leave home at 1 PM and drive their cars in different directions.
How par apart are they at 4 PM?
~
1. A drives at 12 miles per hour westward for two hours and then stops.
2. B drives at 10 miles per hour southwards for one hour, and then drives westwards
at the same speed for three hours.
%
The correct answer is
(C). BOTH statements (1) and (2) TOGETHER are
sufficient to answer the question asked,
but NEITHER statement ALONE is sufficient;
%
#
3
~
How long would it take X to count the video cassettes in a video library ?
~
1. X works twice as fast as Y.
2. Working together, X and Y will complete the job in 10 hours.
%

The correct answer is


(C). BOTH statements (1) and (2) TOGETHER are
sufficient to answer the question asked,
but NEITHER statement ALONE is sufficient;
%
#
5

~
What is the value of a + b + c ?
~
1. a + c = 2b
2. (1/a) + (1/b) + (1/c) = 4/9
%
The correct answer is
(E). Statements (1) and (2) TOGETHER are NOT sufficient
to answer the question asked, and additional data
specific to the problem are needed.
%
#
1
~

One number is five more than half a second number, which is evenly divisible by a
third number. What is the second number ?
~
1. The first number is one less than the second.
2. The third number is two less than half the second.
%
The correct answer is
(A). Statement (1) ALONE is sufficient, but
statement (2) alone is not sufficient
to answer the question asked;
%
#
5
~
Each of 200 electrical switches controls a separate light bulb. How many of the
switches are in the ON position ?
~
1. 50% of the bulbs are glowing.
2. 5% of the bulbs are burnt out.
%
The correct answer is
(E). Statements (1) and (2) TOGETHER are NOT sufficient
to answer the question asked, and additional data

specific to the problem are needed.


%
#
4
~
Each of 200 electrical switches controls a separate light bulb. How many of the
switches are in the ON position if no bulb is fused or burnt ?
~
1. 70% of the bulbs are glowing.
2. 30% of the bulbs are not glowing.
%
The correct answer is
(D). EACH statement ALONE is sufficient
to answer the question asked;
%

#
1
~
Is the product cd positive ?
~
1. 3c = -8*d*d*d
2. d > c + 4

%
The correct answer is
(A). Statement (1) ALONE is sufficient, but
statement (2) alone is not sufficient
to answer the question asked;
%
#
3
~
If x < y, IS (x - y) < yz ?

~
1. y < 0
2. z < 0
%
The correct answer is
(C). BOTH statements (1) and (2) TOGETHER are
sufficient to answer the question asked,
but NEITHER statement ALONE is sufficient;
%
#
5
~
How many people are employed in Company ABC, having an annual payroll bill of $
3,000,000 ?
~
1. Three fourth's of the employees are clerical at an average salary of $ 15,000.
2. With 7% more employees, the payroll would equal $ 3,250,000.
%
The correct answer is
(E). Statements (1) and (2) TOGETHER are NOT sufficient
to answer the question asked, and additional data
specific to the problem are needed.
%
#
4
~
If m = n + 2, what is the value of m*m - 4m + 4 ?
~
1. n = 20
2. n*n = 400
%
The correct answer is
(D). EACH statement ALONE is sufficient
to answer the question asked;
%
#
5
~
An ice-cream shop sells two sizes of ice-cream cones A and B. How many A's were
sold one day ?
~
1. The total sales were $ 300.
2. The A's sold for $ 1 and B's sold for $ 1.50.
%
The correct answer is
(E). Statements (1) and (2) TOGETHER are NOT sufficient
to answer the question asked, and additional data
specific to the problem are needed.
%
#
5
~
An ice-cream shop sells two sizes of ice-cream cones A and B. How many B's were
sold one day ?
~
1. The total sales were $ 300.
2. 100 A's were sold for $ 1 each.
%
The correct answer is
(E). Statements (1) and (2) TOGETHER are NOT sufficient
to answer the question asked, and additional data
specific to the problem are needed.
%
#
5
~
An ice-cream shop sells two sizes of ice-cream cones A and B. How many A's were
sold one day if the total sales that day were $ 300 ?
~
1. The A's sold for $ 1 each.
2. The B's sold for $ 1.50 each.
%
The correct answer is
(E). Statements (1) and (2) TOGETHER are NOT sufficient
to answer the question asked, and additional data
specific to the problem are needed.
%
#
3
~
An ice-cream shop sells two sizes of ice-cream cones A and B. How many A's were
sold one day if the total sales that day were $ 300 ?
~
1. The A's sold for $ 1 each and the B's sold for $ 1.50 each.
2. The total number of B's sold were 100 numbers.
%
The correct answer is
(C). BOTH statements (1) and (2) TOGETHER are
sufficient to answer the question asked,
but NEITHER statement ALONE is sufficient;
%
#
2
~
If a and b are no zeros, is the value of a*a*b positive
~
1. a > 0
2. b > 0
%
The correct answer is
(B). Statement (2) ALONE is sufficient, but statement (1) alone
is not sufficient to answer the question asked;
%
#
5
~
Where is the centre of a circle on the xy plane ?
~
1. The circle passes through both the origin and (0,7).
2. The diameter equals 10.
%
The correct answer is
(E). Statements (1) and (2) TOGETHER are NOT sufficient
to answer the question asked, and additional data
specific to the problem are needed.
%
#
5
~
What is the average of ten numbers ?
~
1. Nine of the numbers sum to 45.
2. One of the numbers is 10.
%
The correct answer is
(E). Statements (1) and (2) TOGETHER are NOT sufficient
to answer the question asked, and additional data
specific to the problem are needed.
%
#
3
~
What is the average of ten numbers ?
~
1. Nine of the numbers sum to 45.
2. The tenth number is 5.
%
The correct answer is
(C). BOTH statements (1) and (2) TOGETHER are
sufficient to answer the question asked,
but NEITHER statement ALONE is sufficient;
%
#
3
~
What is the area of a rectangular field ?
~
1. The diagonal is twice the width.
2. The length is 173 feet.
%
The correct answer is
(C). BOTH statements (1) and (2) TOGETHER are
sufficient to answer the question asked,
but NEITHER statement ALONE is sufficient;
%
#
3
~
What is the area of a rectangular field ?
~
1. The width is half the length.
2. The length is 100 feet.
%
The correct answer is
(C). BOTH statements (1) and (2) TOGETHER are
sufficient to answer the question asked,
but NEITHER statement ALONE is sufficient;
%
#
3
~
What is the perimeter of a rectangular field ?
~
1. The width is half the length.
2. The length is 100 feet.
%
The correct answer is
(C). BOTH statements (1) and (2) TOGETHER are
sufficient to answer the question asked,
but NEITHER statement ALONE is sufficient;
%
#
3
~
What is the perimeter of a rectangular field ?
~
1. The diagonal is twice the width.
2. The length is 173 feet.
%
The correct answer is
(C). BOTH statements (1) and (2) TOGETHER are
sufficient to answer the question asked,
but NEITHER statement ALONE is sufficient;
%
#
3
~
If 2*a*a + b*b = 27, what is the value of a ?
~
1. b is positive
2. a = b
%
The correct answer is
(C). BOTH statements (1) and (2) TOGETHER are
sufficient to answer the question asked,
but NEITHER statement ALONE is sufficient;
%
#
3
~
If 2a + b = 27, what is the value of a ?
~
1. b is positive
2. a = b
%
The correct answer is
(C). BOTH statements (1) and (2) TOGETHER are
sufficient to answer the question asked,
but NEITHER statement ALONE is sufficient;
%
#
2
~
How far is City B from City A ?
~
1. A car that gives a mileage of 30 miles per gallon drove the distance on exactly
one tank of gasoline.
2. Another car averaged 60 miles per hour and arrived in 6 hours.
%
The correct answer is
(B). Statement (2) ALONE is sufficient, but statement (1) alone
is not sufficient to answer the question asked;
%
#
4
~
How far is City B from City A ?
~
1. Car Y averaged 90 miles per hour and arrived in 4 hours.
2. Car X averaged 60 miles per hour and arrived in 6 hours.
%
The correct answer is
(D). EACH statement ALONE is sufficient
to answer the question asked;
%
#
5
~
How far is City B from City A ?
~
1. A car that gives a mileage of 30 miles per gallon drove the distance on exactly
one tank of gasoline.
2. Another car that gives a mileage of 40 miles per gallon drove the distance on
exactly one tank of gasoline.
%
The correct answer is
(E). Statements (1) and (2) TOGETHER are NOT sufficient
to answer the question asked, and additional data
specific to the problem are needed.
%
#
5
~
Is the value xy > 27 ?
~
1. 2 < x < 5
2. 6 > y
%
The correct answer is
(E). Statements (1) and (2) TOGETHER are NOT sufficient
to answer the question asked, and additional data
specific to the problem are needed.
%
#
1
~
Three chef's X, Y and Z know the secret recipe of 19 different and rare dishes.
There is no overlap of information. How many secret recipes does Y know ?
~
1. X know one more than Y and twice as many as Z.
2. Y knows three more secret recipes than Z.
%
The correct answer is
(A). Statement (1) ALONE is sufficient, but
statement (2) alone is not sufficient
to answer the question asked;
%
#
1
~
Three chef's X, Y and Z know the secret recipe of 19 different and rare dishes.
There is no overlap of information. How many secret recipes does Y know ?
~
1. X know one more than Y and twice as many as Z.
2. X knows the maximum number of secret recipes.
%
The correct answer is
(A). Statement (1) ALONE is sufficient, but
statement (2) alone is not sufficient
to answer the question asked;
%
#
5
~
Three chef's X, Y and Z know the secret recipe of 19 different and rare dishes.
There is no overlap of information. How many secret recipes does Y know ?
~
1. Z knows the minimum number of the secret recipes.
2. X knows maximum number of the secret recipes.
%
The correct answer is
(E). Statements (1) and (2) TOGETHER are NOT sufficient
to answer the question asked, and additional data
specific to the problem are needed.
%
#
1
~
C is an positive integer and is a perfect cube. What is the value of C ?
~
1. 30 < C < 100
2. C is an even number
%
The correct answer is
(A). Statement (1) ALONE is sufficient, but
statement (2) alone is not sufficient
to answer the question asked;
%
#
3
~
C is an positive integer and is a perfect cube. What is the value of C ?
~
1. C is less than 10.
2. C is an even number
%
The correct answer is
(C). BOTH statements (1) and (2) TOGETHER are
sufficient to answer the question asked,
but NEITHER statement ALONE is sufficient;
%
#
3
~
C is an positive integer and is a perfect cube. What is the value of C ?
~
1. C is more than 2 but less than 30.
2. C is an odd number
%
The correct answer is
(C). BOTH statements (1) and (2) TOGETHER are
sufficient to answer the question asked,
but NEITHER statement ALONE is sufficient;
%
#
1
~
C is an positive integer and is a perfect cube. What is the value of C ?
~
1. 100 < C < 130
2. C is an even number
%
The correct answer is
(A). Statement (1) ALONE is sufficient, but
statement (2) alone is not sufficient
to answer the question asked;
%
#
3
~
How much gross profit did Store X make during its Christmas sale ?
~
1. The store X sold 100 videos for $ 38,000.
2. The average cost price of an video was $ 175.
%
The correct answer is
(C). BOTH statements (1) and (2) TOGETHER are
sufficient to answer the question asked,
but NEITHER statement ALONE is sufficient;
%
#
5
~
What is the volume of a cylindrical can ?
~
1. The radius of the can is 3 inches.
2. The area of the bottom is 9pi square inches.
%
The correct answer is
(E). Statements (1) and (2) TOGETHER are NOT sufficient
to answer the question asked, and additional data
specific to the problem are needed.
%
#
3
~
What is the volume of a cylindrical can ?
~
1. The radius of the can is 3 inches.
2. The height of the can is 9 inches.
%
The correct answer is
(C). BOTH statements (1) and (2) TOGETHER are
sufficient to answer the question asked,
but NEITHER statement ALONE is sufficient;
%
#
3
~
What is the volume of a cylindrical can ?
~
1. The height of the can is 9 inches.
2. The area of the bottom is 9pi square inches.
%
The correct answer is
(C). BOTH statements (1) and (2) TOGETHER are
sufficient to answer the question asked,
but NEITHER statement ALONE is sufficient;
%
#
3
~
What is the volume of a cylindrical can ?
~
1. The height of the can is twice the radius of the circular bottom.
2. The area of the bottom is 9pi square inches.
%
The correct answer is
(C). BOTH statements (1) and (2) TOGETHER are
sufficient to answer the question asked,
but NEITHER statement ALONE is sufficient;
%
#
1
~
What is Robert's typing speed in words per minute ?
~
1. Robert completed typing a report of 2400 words in one hour.
2. The report has 8 pages each having an average of 300 words.
%
The correct answer is
(A). Statement (1) ALONE is sufficient, but
statement (2) alone is not sufficient
to answer the question asked;
%
#
3
~
What is Robert's typing speed in words per minute ?
~
1. Robert completed the report X in one hour.
2. The report X has 8 pages each having an average of 300 words.
%
The correct answer is
(C). BOTH statements (1) and (2) TOGETHER are
sufficient to answer the question asked,
but NEITHER statement ALONE is sufficient;
%
#
5
~
What is Robert's typing speed in words per minute ?
~
1. Robert completed the report X in one hour.
2. The report X has 8 pages.
%
The correct answer is
(E). Statements (1) and (2) TOGETHER are NOT sufficient
to answer the question asked, and additional data
specific to the problem are needed.
%
#
5
~
What is Robert's typing speed in words per minute ?
~
1. Susan completed the report X in one hour.
2. The report X has 8 pages each having an average of 300 words.
%
The correct answer is
(E). Statements (1) and (2) TOGETHER are NOT sufficient
to answer the question asked, and additional data
specific to the problem are needed.
%
#
5
~
Which is greater, cf or fg ?
~
1. c > g
2. f*f = cg
%
The correct answer is
(E). Statements (1) and (2) TOGETHER are NOT sufficient
to answer the question asked, and additional data
specific to the problem are needed.
%
#
5
~
Which is greater, ab or bc ?
~
1. a > c
2. b*b = ac
%
The correct answer is
(E). Statements (1) and (2) TOGETHER are NOT sufficient
to answer the question asked, and additional data
specific to the problem are needed.
%
#
5
~
Which is greater, xy or yz ?
~
1. x > z
2. y*y = xz
%
The correct answer is
(E). Statements (1) and (2) TOGETHER are NOT sufficient
to answer the question asked, and additional data
specific to the problem are needed.
%
#
3
~
What is the value of a + b ?
~
1. 2c - 7 = 0
2. a + b - 3c = 11
%
The correct answer is
(C). BOTH statements (1) and (2) TOGETHER are
sufficient to answer the question asked,
but NEITHER statement ALONE is sufficient;
%
#
3
~
What is the value of x + y ?
~
1. 2z - 7 = 0
2. x + y - 3z = 11
%
The correct answer is
(C). BOTH statements (1) and (2) TOGETHER are
sufficient to answer the question asked,
but NEITHER statement ALONE is sufficient;
%
#
5
~
What is the value of a + b ?
~
1. 2c - 7 = 0
2. a + b - 3d = 11
%
The correct answer is
(E). Statements (1) and (2) TOGETHER are NOT sufficient
to answer the question asked, and additional data
specific to the problem are needed.
%
#
5
~
What is the value of a + b ?
~
1. 2c - 7 = 0
2. a + b - 4d = 11
%
The correct answer is
(E). Statements (1) and (2) TOGETHER are NOT sufficient
to answer the question asked, and additional data
specific to the problem are needed.
%
#
3
~
What is the volume of a certain box B?
~
1. One side of the box has an area of 25 square inches.
2. The box is cubical.
%
The correct answer is
(C). BOTH statements (1) and (2) TOGETHER are
sufficient to answer the question asked,
but NEITHER statement ALONE is sufficient;
%
#
5
~
What is the volume of a certain box B?
~
1. One side of the box has an area of 25 square inches.
2. The box is not cubical.
%
The correct answer is
(E). Statements (1) and (2) TOGETHER are NOT sufficient
to answer the question asked, and additional data
specific to the problem are needed.
%
#
3
~
What is the volume of a certain box B?
~
1. One side of the box has an area of 25 square inches.
2. The height of the box is 10 inches.
%
The correct answer is
(C). BOTH statements (1) and (2) TOGETHER are
sufficient to answer the question asked,
but NEITHER statement ALONE is sufficient;
%
#
3
~
What is the volume of a certain box B?
~
1. One side of the box has a dimension of 5 inches.
2. The box is cubical.
%
The correct answer is
(C). BOTH statements (1) and (2) TOGETHER are
sufficient to answer the question asked,
but NEITHER statement ALONE is sufficient;
%
#
5
~
What is the volume of a certain box B?
~
1. One side of the box has an area of 25 square inches.
2. The box is red in color.
%
The correct answer is
(E). Statements (1) and (2) TOGETHER are NOT sufficient
to answer the question asked, and additional data
specific to the problem are needed.
%
#
4
~
What is the volume of a certain cubical box B?
~
1. One side of the box has an area of 25 square inches.
2. The box has a side dimension of 5 inches.
%
The correct answer is
(D). EACH statement ALONE is sufficient
to answer the question asked;
%
#
4
~
Given that x = y + z, where the three values are different positive integers, is x
a prime number ?
~
1. y and z are odd.
2. z = 3y
%
The correct answer is
(D). EACH statement ALONE is sufficient
to answer the question asked;
%
#
4
~
Given that a = b + c, where the three values are different positive integers, is
'a' a prime number ?
~
1. b and c are odd.
2. c = 3b
%
The correct answer is
(D). EACH statement ALONE is sufficient
to answer the question asked;
%
#
1
~
What is the distance from City A to City B ?
~
1. A jet takes 4 hours to reach at an average speed of 600 miles per hour.
2. The jet consumes 4 gallons of fuel per mile.
%
The correct answer is
(A). Statement (1) ALONE is sufficient, but
statement (2) alone is not sufficient
to answer the question asked;
%
#
4
~
What is the distance from City A to City B ?
~
1. A jet X takes 4 hours to reach at an average speed of 600 miles per hour.
2. Another jet Y takes 3 hours to reach at an average speed of 800 miles per hour.
%
The correct answer is
(D). EACH statement ALONE is sufficient
to answer the question asked;
%
#
5
~
What is the distance from City A to City B ?
~
1. A jet X consumes 3 gallons of fuel per mile.
2. Another jet Y consumes 4 gallons of fuel per mile.
%
The correct answer is
(E). Statements (1) and (2) TOGETHER are NOT sufficient
to answer the question asked, and additional data
specific to the problem are needed.
%
#
3
~
What is the distance from City A to City B ?
~
1. A jet X takes 4 hours to reach the destination City B from City A.
2. The Jet A flies at an average speed of 600 miles per hour.
%
The correct answer is
(C). BOTH statements (1) and (2) TOGETHER are
sufficient to answer the question asked,
but NEITHER statement ALONE is sufficient;
%
#
5
~
Is A greater than B in duration ?
~
1. There are 48 B's each day.
2. A is longer than a minute.
%
The correct answer is
(E). Statements (1) and (2) TOGETHER are NOT sufficient
to answer the question asked, and additional data
specific to the problem are needed.
%
#
3
~
Is A greater than B in duration ?
~
1. There are 48 B's each day.
2. There are 24 A's in each day.
%
The correct answer is
(C). BOTH statements (1) and (2) TOGETHER are
sufficient to answer the question asked,
but NEITHER statement ALONE is sufficient;
%
#
4
~
What is the vale of m?
~
1. 4m - 5 = 7
2. m*m + 9 = 6m
%
The correct answer is
(D). EACH statement ALONE is sufficient
to answer the question asked;
%
#
4
~
What is the vale of m?
~
1. 4m - 5 = 7
2. m + 9 = 12
%
The correct answer is
(D). EACH statement ALONE is sufficient
to answer the question asked;
%
#
4
~
What is the vale of m?
~
1. 5m - 7 = 8
2. m + 9 = 12
%
The correct answer is
(D). EACH statement ALONE is sufficient
to answer the question asked;
%
#
3
~
How much antifreeze must be mixed with water to fill a car radiator ?
~
1. The radiator has a capacity of 3 gallons.
2. The solution must be 40% water.
%
The correct answer is
(C). BOTH statements (1) and (2) TOGETHER are
sufficient to answer the question asked,
but NEITHER statement ALONE is sufficient;
%
#
5
~
How much antifreeze must be mixed with water to fill a car radiator ?
~
1. The solution must be 60% antifreeze.
2. The solution must be 40% water.
%
The correct answer is
(E). Statements (1) and (2) TOGETHER are NOT sufficient
to answer the question asked, and additional data
specific to the problem are needed.
%
#
3
~
How much antifreeze must be mixed with water to fill a car radiator ?
~
1. The radiator has a capacity of 3 gallons ?
2. The ratio of antifreeze to water should be 3 to 2.
%
The correct answer is
(C). BOTH statements (1) and (2) TOGETHER are
sufficient to answer the question asked,
but NEITHER statement ALONE is sufficient;
%
#
5
~
If a is to b as b is to c, is c positive ?
~
1. c = 2a
2. b*b = 32
%
The correct answer is
(E). Statements (1) and (2) TOGETHER are NOT sufficient
to answer the question asked, and additional data
specific to the problem are needed.
%
#
1
~
What is the sum of five numbers ?
~
1. The average of the numbers is zero.
2. Only one of the numbers is positive.
%
The correct answer is
(A). Statement (1) ALONE is sufficient, but
statement (2) alone is not sufficient
to answer the question asked;
%
#
5
~
Is x = y ?
~
1. 4x - 2y = 80
2. x = (40+y) /2
%
The correct answer is
(E). Statements (1) and (2) TOGETHER are NOT sufficient
to answer the question asked, and additional data
specific to the problem are needed.
%
#
5
~
Is x = y ?
~
1. 2x - y = 40
2. x = (40+y) /2
%
The correct answer is
(E). Statements (1) and (2) TOGETHER are NOT sufficient
to answer the question asked, and additional data
specific to the problem are needed.
%
#
5
~
Is x = y ?
~
1. 12x - 6y = 240
2. x = (40+y) /2
%
The correct answer is
(E). Statements (1) and (2) TOGETHER are NOT sufficient
to answer the question asked, and additional data
specific to the problem are needed.
%
#
1
~
Is the number n an integer ?
~
1. The factors of N are 3, 5 and 7.
2. N cannot be evenly divided by 2.
%
The correct answer is
(A). Statement (1) ALONE is sufficient, but
statement (2) alone is not sufficient
to answer the question asked;
%
#
5
~
What is the orientation of a line in the xy plane ?
~
1. The line passes through the point (5,b).
2. b = -3
%
The correct answer is
(E). Statements (1) and (2) TOGETHER are NOT sufficient
to answer the question asked, and additional data
specific to the problem are needed.
%
#
2
~
How much principal and interest are still owed on a home originally sold for $
179,000.
~
1. The interest rate when the home was purchased was 9%.
2. The owner has yet to make 70 payments of 1124 each.
%
The correct answer is
(B). Statement (2) ALONE is sufficient, but statement (1) alone
is not sufficient to answer the question asked;
%
|

Vous aimerez peut-être aussi